Dear Professor Zeilberger, This email is in response to your recent post on the arxiv on the number of partitions of n into distinct parts. I believe I have a proof that: log(f(n)) ~ (6n)^{1/3} log(n)/3. The basic idea is as follows. A partition into parts with distinct multiplicities can be specified by the following data: * A non-negative m equal to the number of distinct sizes of parts * A set P of positive integers of size m corresponding to the sizes of parts * A set M of positive integers of size m corresponding to the multiplicities of the parts * A bijection pi: P -> M. The partition has pi(p) parts of size p for each p in P. Thus the size of the total partition is sum p* pi(p). It is not hard to see that m = O(n^{1/3}) since otherwise the size would be forced to be too big. It is also not hard to show that there are only 2^{O(n^{1/3})} ways of picking M and P so that is it possible to obtain a partition of size at most n, thus we are left with finding the maximum possible number of pi given a single P and M. It should be noted that the smallest possible partition for a given m has m+1-k parts of size k and has total size m(m+1)(m+2)/6. Thus m <= (6n)^{1/3}, and the number of possible pi is at most [(6n)^{1/3}]! On the other hand we can show that if eps >0 and m < (6n)^{1/3}(1-eps) then for P=M={1,2,...,m}, a reasonable fraction of the m! possible pi. In particular if |pi(k) + k - m| < delta m for all k and some small delta, our partition should be small enough, but will leave about (delta m)^m possible permutations, which gives the correct asymptotic for log(f(n)). I was puzzled for a while about why the numerical data seemed to suggest the asymptotic of C sqrt(n). I can point to two reasons why the data for n <= 508 should fit this better than my asymptotic. 1) Considering the ratio [ n^{1/3} log(n) ] / [ n^{1/2} ], we find that it obtains and absolute maximum near n=400. This means that the ratio of the real asymptotic and sqrt(n) should be fairly flat around this neighborhood. 2) I expect that my asymptotic has a relative error on the order of log(n)^{-0.5}. For n=500, this is about 0.6, certainly big enough to account for the 20% discrepancy between log(f(508)) and the predicted value. -Daniel Kane